Search found 520 matches


Awesome score eagleeye! A Question: How reliable did you find Manhattan tests? As I have given two tests till now and scored 600(Q42,V32) and 570(Q41,v28), which are way away from my target score(750+). Personally, while solving I felt that Quants section was tough in MGMAT tests than questions in ...

by eagleeye

Tue Sep 11, 2012 6:56 pm
Forum: I just Beat The GMAT!
Topic: 770 (Q50,V47): How BEATtheGMAT helped me beat the GMAT!!
Replies: 21
Views: 10383

Thank you all for the kind wishes and the congrats! I am glad you found the debrief useful. Good luck in your journey!

by eagleeye

Thu Sep 06, 2012 5:31 pm
Forum: I just Beat The GMAT!
Topic: 770 (Q50,V47): How BEATtheGMAT helped me beat the GMAT!!
Replies: 21
Views: 10383

I just got the official score email. Here's the final score: (V47/99%) (Q50/90%) (Overall 770/99%) (AWA 6.0/90%) (IR: 7/82%) As promised, I am writing my GMAT journey and preparation tips. THE MBA JOURNEY Its going to be a long post so get your granola bars and your gatorade and hang on tight. My jo...

by eagleeye

Tue Sep 04, 2012 8:30 pm
Forum: I just Beat The GMAT!
Topic: 770 (Q50,V47): How BEATtheGMAT helped me beat the GMAT!!
Replies: 21
Views: 10383

770 (Q50,V47): How BEATtheGMAT helped me beat the GMAT!!

Update :As promised I have posted the updated score and debrief in the link below. ) http://www.beatthegmat.com/770-q50-v47-how-beatthegmat-helped-me-beat-the-gmat-t117589-15.html#498518 I just finished my GMAT. Needless to say that I am pretty thrilled. I wanted to take this opportunity to thank a...

by eagleeye

Sat Aug 18, 2012 2:44 pm
Forum: I just Beat The GMAT!
Topic: 770 (Q50,V47): How BEATtheGMAT helped me beat the GMAT!!
Replies: 21
Views: 10383

Hey eagleeye, Here, we are only given that X is a positivenumber. How can you say that Squareroot(x) is also positive ?? Correct me if I m wrong. Please read this post. It might help. sqrt(x) is never negative for a real number. http://www.beatthegmat.com/squares-square-roots-inequalities-and-modul...

by eagleeye

Tue Aug 14, 2012 9:45 am
Forum: Data Sufficiency
Topic: help!
Replies: 6
Views: 1328

if x is a positive integer, is sqrt{x} < 2.5x - 5 ? 1. x < 3 2. x is a prime number Let's rephrase the question as Is sqrt(x) + 5 - 2.5x < 0 Given: x is a positive integer. 1. x < 3 Since x is a positive integer, x can either be 1 or 2. For x=1, sqrt(1) + 5 -2.5 > 0 For x=2, sqrt(2) + 5 -2.5*2 > 0....

by eagleeye

Sun Aug 12, 2012 4:03 pm
Forum: Data Sufficiency
Topic: help!
Replies: 6
Views: 1328

grandh01 wrote:sorry about that I made a mistake, I edited the post.
And I've edited mine :)

by eagleeye

Sat Aug 11, 2012 3:18 pm
Forum: Problem Solving
Topic: fractions
Replies: 3
Views: 1150

In Township K, 1/5 of the housing units are equipped with cable television. If 1/10 of the housing units, including 1/2 of those that are equipped with cable television, are equipped with videocassette recorders, what fraction of the housing units have neither cable television nor videocassette rec...

by eagleeye

Sat Aug 11, 2012 2:11 pm
Forum: Problem Solving
Topic: fractions
Replies: 3
Views: 1150

Hello Experts, Can you please clarify, how can we know the age of the tombs if we know age of the trees. Agreed if we know rings and other details we can know the age of the logs at the time they were cut, but how we will know the age of the tombs as I assume once the logs are cut no rings are form...

by eagleeye

Sat Aug 11, 2012 10:49 am
Forum: Critical Reasoning
Topic: LSAT CR Q
Replies: 13
Views: 4426

mcdesty wrote:
(-1-7) is -8 and that is 8 away from zero not 6...X and Y can't be -1 and -7...
-1, -7
(-1) - (-7) = -1 + 7 = -6.

by eagleeye

Sat Aug 11, 2012 10:32 am
Forum: Data Sufficiency
Topic: What is the value of (x-y)^4
Replies: 9
Views: 3778

Experts, I had marked the answer of attached question as C as there can be many numbers from A and B that satisfy them but when we combine them, we get the common number satisfying both.Not sure how the answer is something else. Plz help. We need to find the LCM of x,6 and 9. With that in mind, con...

by eagleeye

Fri Aug 10, 2012 2:46 pm
Forum: GMAT Math
Topic: Least common mutiple- GMAT Prep
Replies: 3
Views: 1577

Though, could solve it but would like to know what will be the strategy adopted by Geniuses out there.Kindly share. This question is testing what you know about primes and counting. Statement 1 talks about 100 primes between 1 and p+1. Since we know that p+1 is not a prime, and we can find the 98 p...

by eagleeye

Fri Aug 10, 2012 2:19 pm
Forum: GMAT Math
Topic: Prime # question- GMATPrep
Replies: 4
Views: 1505

GMAT Experts, Not sure how to arrive at the correct answer for attached question. Comparison by "that" seems bit odd but that happens to be the correct answer. As with any SC question, this one requires us to pay attention to the non-underlined part. We'll eliminate options one by one and...

by eagleeye

Fri Aug 10, 2012 11:07 am
Forum: Sentence Correction
Topic: SC question Comparison- GMAT Prep
Replies: 2
Views: 1121

GMAT Experts, Can you plz confirm the right way to approach the attached question.Assumed #s to solve it but just checking for a better/shorter method if any. I would do it this way. First of all we need to realize that its an inequality question which is asking us. Is least value of set A < least ...

by eagleeye

Fri Aug 10, 2012 9:56 am
Forum: Problem Solving
Topic: GMAT Prep DI question- Experts please reply ASAP
Replies: 3
Views: 1003

GMAT Experts, Can you plz confirm the best method to approach question as attached below. Please share steps to arrive at correct answer. Both Barker and Carey would be really proud of this question. Come on down! Jokes apart, the way I did it is as follows. See the attached figure. Now, the A, B1,...

by eagleeye

Thu Aug 09, 2012 9:28 pm
Forum: GMAT Math
Topic: Probability GMAT Prep - plz confirm the correct approach
Replies: 4
Views: 2264